LSAT and Law School Admissions Forum

Get expert LSAT preparation and law school admissions advice from PowerScore Test Preparation.

 Administrator
PowerScore Staff
  • PowerScore Staff
  • Posts: 8918
  • Joined: Feb 02, 2011
|
#38550
Complete Question Explanation
(The complete setup for this game can be found here: lsat/viewtopic.php?t=15133)

The correct answer choice is (A)

This Justify question asks us to identify a rule that, if true, would fully determine the placement of all six variables. While a prephrase is difficult to formulate, it is important to know what we do not know. For instance, we do not know anything about L: as discussed in Question #21, L can be in either group. To justify a single solution to this game, the correct answer choice must settle this uncertainty—a simple insight that helps eliminate answer choice (B) without further testing.

Had you relied on the used of templates in your initial analysis, this question would be quite simple: simply look for an answer choice that 1) establishes the placement of L; and 2) forces the solution to conform to Template 1, which leaves no room for uncertainty. Answer choice (A) is the only answer that satisfies both objectives.

Answer choice (A) is the correct answer choice. If K is published in the fall, then O must also be published in the fall (third rule), along with N (second rule). However, if N is published in the fall, by the contrapositive of the fourth rule we know that M must be published in the spring, which, in turn, requires P to be published in the fall (first rule). The placement of all five variables is settled. Answer choice (A) also specifies that L is published in the spring, ensuring that the placement of all six variables is fully determined.

Answer choice (B) is incorrect, because the placement of L is still unknown.

Answer choice (C) is incorrect, because if P were published in the fall, all we would know is that M is published in the spring. No other deductions can be drawn about the remaining four variables (to make an inference about N based on the fourth rule would be a Mistaken Negation of that rule).

Answer choice (D) is incorrect. If K is published in the spring, all we know is that N must also be published in the spring (second rule). No inference about M can be drawn on the basis of this information (to conclude otherwise would be a Mistaken Reversal of the last rule).

Answer choice (E) is attractive, but incorrect. If M is published in the fall, then P must be published in the spring (first rule) along with N (fourth rule). Thanks to the second rule, K must also be published in the spring. Unfortunately, that still does not tell us anything about O, which can be published in either season.
 FK00144
  • Posts: 12
  • Joined: Dec 12, 2018
|
#63700
based on the template L could go in Fall or Spring when K is placed in the Fall, so how does A) determine L is in the spring and not fall. Hope that made sense
 Adam Tyson
PowerScore Staff
  • PowerScore Staff
  • Posts: 5158
  • Joined: Apr 14, 2011
|
#63842
You're exactly right about that template, FK00144, but read that answer choice again - it's not saying that IF K is in the Fall, THEN L must be in the Spring. It's saying IF K is in the Fall AND L is in the Spring. If both of those things happen (and all of the answers are about two things happening, not just one), then there is only one solution to the game. K in the Fall forces both N and O to be in the Fall; N in the Fall forces M into the Spring, which in turn forces P into the Fall. This isn't about K in the Fall forcing L to go to the Spring, but about what happens when K is in the Fall and L is in the Spring. That solves the game, and that's what we are looking for here!

Get the most out of your LSAT Prep Plus subscription.

Analyze and track your performance with our Testing and Analytics Package.